Diễn Đàn MathScopeDiễn Đàn MathScope
  Diễn Đàn MathScope
Ghi Danh Hỏi/Ðáp Thành Viên Social Groups Lịch Ðánh Dấu Ðã Ðọc

Go Back   Diễn Đàn MathScope > Sơ Cấp > Việt Nam và IMO

News & Announcements

Ngoài một số quy định đã được nêu trong phần Quy định của Ghi Danh , mọi người tranh thủ bỏ ra 5 phút để đọc thêm một số Quy định sau để khỏi bị treo nick ở MathScope nhé !

* Nội quy MathScope.Org

* Một số quy định chung !

* Quy định về việc viết bài trong diễn đàn MathScope

* Nếu bạn muốn gia nhập đội ngũ BQT thì vui lòng tham gia tại đây

* Những câu hỏi thường gặp

* Về việc viết bài trong Box Đại học và Sau đại học


Trả lời Gởi Ðề Tài Mới
 
Ðiều Chỉnh Xếp Bài
Old 08-08-2010, 05:19 PM   #1
huynhcongbang
Administrator

 
huynhcongbang's Avatar
 
Tham gia ngày: Feb 2009
Đến từ: Ho Chi Minh City
Bài gởi: 2,413
Thanks: 2,165
Thanked 4,188 Times in 1,381 Posts
Gửi tin nhắn qua Yahoo chát tới huynhcongbang
Đề thi và lời giải đề TST 2005

Mình xin gửi các bạn lời giải đề thi chọn đội tuyển QG của Việt Nam dự thi IMO 2005. Về nội dung, có thể nhận xét sơ lược như sau:

* Bài toán 1 là sự kết hợp của hai bổ đề quen thuộc:
+ Tâm vị tự ngoài của ba đường tròn phân biệt thì thẳng hàng.
+ Cho tam giác ABC nội tiếp O và D, E, F là tiếp điểm của đường tròn nội tiếp I lên các cạnh tam giác. Khi đó, trực tâm của tam giác DEF nằm trên IO.
Chứng minh được hai bổ đề này thì bài toán đã giải quyết được hơn 90%.
Có thể với một số bạn, hai bổ đề trên chưa quen nhưng khi giải bài này sẽ phát hiện ra sự cần thiết của việc chứng minh nó khi giải bài toán.
Bổ đề 2 ở trên có thể chứng minh bằng nhiều cách nhưng mình nghĩ dùng phép nghịch đảo là nhanh nhất.

* Bài toán 2 là một bài tổ hợp dùng song ánh và các tính toán phân hoạch thông dụng trên tập hợp; ngoài cách này ra, khó có thể dùng các lập luận đơn thuần dùng các công thức tổ hợp, chỉnh hợp, hoán vị,... để tìm được kết quả đó.

* Bài toán 3 là một bài PT hàm có vẻ bề ngoài đơn giản và thực tế nếu cho $f(x) $ liên tục trên tập số thực thì bài toán trở nên bình thường; nhưng ở đây chỉ thu hẹp $f(x) $ trên tập số nguyên thì bài toán không còn đơn giản thế nữa. Mấu chốt của bài toán là cần tìm một đẳng thức biểu diễn lập phương của mỗi số nguyên bất kì thành 5 tổng lập phương của các số nguyên khác. Tiếp đó là tính giá trị của $f(x) $ tại các điểm $\pm 1,\pm2,... $ rồi quy nạp lên. Có thể nói, ngoài cách tìm đẳng thức đã nêu ở trên ra, bài toán này không còn cách giải quyết nào khác.

*Bài toán 4: đây là một BĐT ba biến trông rất đẹp mắt và cũng không quá khó; bằng cách đặt ẩn phụ dưới dạng tỉ số quen thuộc là có thể chuyễn nó về một BĐT đối xứng. Lời giải ở trên dùng cách chứng minh được gợi ý trong cuốn "Sáng tạo BĐT" của Phạm Kim Hùng (người trực tiếp tham gia kì thi TST 2005 này), ngoài ra có thể dùng dồn biến cũng đánh giá biểu thức này không khó.

*Bài toán 5:: bài này thể hiện sự phát triển từ hai định lí quen thuộc trong số chính phương có mod là:
$\left ( \frac{-1}{p} \right )=(-1)^{\frac{p-1}{4}}(modp),\left ( \frac{2}{p} \right )=(-1)^{\frac{p^2-1}{8}}(modp) $.
Chứng minh được hai định lí này thì chỉ cần dùng thêm vài phép biến đổi trên phần nguyên là ra được kết quả cần tính, chú ý đẳng thức quen thuộc là:
$[2x]-2[x]=1,0\le{x}<\frac{1}{2}\vee [2x]-2[x]=0,\frac{1}{2}\le x<1 $.

*Bài toán 6:: bài này đưa ra dưới một hình thức lạ và dường như trong tính chất số học của đề bài, còn có liên quan đến vấn đề giới hạn của giải tích. Muốn giải bài này cũng cần chứng minh thêm một bổ đề quan trọng và không dễ thấy được. Vì thế dù cùng là một bài số học nhưng mức độ của nó rõ ràng là khó hơn bài toán 5.

Nhận xét chung thì đề bài này có hình thức đẹp nhưng vì cũng chỉ xuất phát từ các vấn đề cũ để phát biểu thành bài toán nên chưa hay lắm!
Có lẽ trong lịch sử thi chọn đội tuyển của Việt Nam thì đề TST 2005 là nổi tiếng nhất với nhiều vấn đề liên quan đến nó mà chắc mọi người ai cũng biết nên mình không nêu ra ở đây.
Trong lời giải này, mình tham khảo gợi ý từ mathlinks.ro, chuyên đề tổng hợp của diendantoanhoc, sách "Sáng tạo BĐT" tác giả Phạm Kim Hùng, file lời giải đề TST từ 1990 đến 2004 (bài số 6 đề TST 1997).
Bài số 5 mình đang chờ các bạn chứng minh giúp một định lí trong lí thuyết số chính phương có modun nên chưa giải đầy đủ được, chỉ mới áp dụng nó vào bài giải thôi.

Mong rằng lời giải này sẽ giúp các bạn có được một tài liệu tham khảo nhỏ!
[RIGHT][I][B]Nguồn: MathScope.ORG[/B][/I][/RIGHT]
 
File Kèm Theo
Kiểu File : pdf De TST 2005.pdf (88.7 KB, 365 lần tải)
Kiểu File : pdf LoiGiaiVietNamTST2005.pdf (255.8 KB, 622 lần tải)

thay đổi nội dung bởi: huynhcongbang, 08-08-2010 lúc 07:54 PM
huynhcongbang is offline   Trả Lời Với Trích Dẫn
The Following 12 Users Say Thank You to huynhcongbang For This Useful Post:
22021993hh (12-08-2010), Conan Edogawa (08-08-2010), huuphuc (13-08-2010), king_math96 (09-08-2010), leviethai (12-08-2010), luatdhv (08-08-2010), nam1994 (08-08-2010), nguyenxuanhuy (12-08-2010), pHnAM (08-08-2010), phuongloan (10-08-2010), quocbaoct10 (18-11-2013), uzuhaku (03-01-2012)
Old 08-08-2010, 07:06 PM   #2
novae
+Thành Viên Danh Dự+
 
novae's Avatar
 
Tham gia ngày: Jul 2010
Đến từ: Event horizon
Bài gởi: 2,453
Thanks: 53
Thanked 3,057 Times in 1,288 Posts
Lời giải bài 3 thiếu trường hợp như sau:
Trích:
$f(1)=f^3(1)\Leftrightarrow f(1)=1\vee f(1)=0 $
chính xác phải là $f(1)=\pm 1\vee f(1)=0 $
dẫn đến thiếu nghiệm $f(x)=-x, \forall x\in \mathbb{Z} $
[RIGHT][I][B]Nguồn: MathScope.ORG[/B][/I][/RIGHT]
 
__________________
M.
novae is offline   Trả Lời Với Trích Dẫn
Old 08-08-2010, 07:55 PM   #3
huynhcongbang
Administrator

 
huynhcongbang's Avatar
 
Tham gia ngày: Feb 2009
Đến từ: Ho Chi Minh City
Bài gởi: 2,413
Thanks: 2,165
Thanked 4,188 Times in 1,381 Posts
Gửi tin nhắn qua Yahoo chát tới huynhcongbang
Đã update lại file rồi, cảm ơn mọi người đã ủng hộ, đã đọc bài này và góp ý dùm!
[RIGHT][I][B]Nguồn: MathScope.ORG[/B][/I][/RIGHT]
 

thay đổi nội dung bởi: huynhcongbang, 08-08-2010 lúc 10:00 PM
huynhcongbang is offline   Trả Lời Với Trích Dẫn
Old 08-08-2010, 09:04 PM   #4
herr.casanova
+Thành Viên+
 
Tham gia ngày: Aug 2010
Bài gởi: 33
Thanks: 17
Thanked 33 Times in 13 Posts
Mình có một cách sơ cấp hơn cho việc chứng mình $\frac{1}{1}+\frac{1}{2}+\dots $ tiến tới vô cùng như sau:

$\frac{1}{1} + \frac{1}{2} + \dots +\frac{1}{9} > \frac{9}{9} > \frac{9}{10} $

$\frac{1}{10} + \dots + \frac{1}{99} > \frac{90}{99} > \frac{9}{10} $
...
Hay tổng quát:
$\frac{1}{10^n}+ \dots + \frac{1}{10^{n+1}-1} > \frac{9\times 10^n}{10^{n+1}-1} > \frac{9}{10} $
Cho $n $ chạy từ $0 $ đến $\infty $ ta sẽ có điều phải chứng minh
[RIGHT][I][B]Nguồn: MathScope.ORG[/B][/I][/RIGHT]
 

thay đổi nội dung bởi: herr.casanova, 08-08-2010 lúc 09:06 PM
herr.casanova is offline   Trả Lời Với Trích Dẫn
The Following 2 Users Say Thank You to herr.casanova For This Useful Post:
huynhcongbang (08-08-2010), xtungftu (21-08-2011)
Old 12-08-2010, 06:40 PM   #5
dduclam
+Thành Viên Danh Dự+
 
dduclam's Avatar
 
Tham gia ngày: Nov 2007
Đến từ: Đại học Sư phạm Hà Nội
Bài gởi: 481
Thanks: 63
Thanked 168 Times in 92 Posts
Gửi tin nhắn qua Yahoo chát tới dduclam
Trích:
Nguyên văn bởi herr.casanova View Post
Mình có một cách sơ cấp hơn cho việc chứng mình $\frac{1}{1}+\frac{1}{2}+\dots $ tiến tới vô cùng như sau:

$\frac{1}{1} + \frac{1}{2} + \dots +\frac{1}{9} > \frac{9}{9} > \frac{9}{10} $

$\frac{1}{10} + \dots + \frac{1}{99} > \frac{90}{99} > \frac{9}{10} $
...
Hay tổng quát:
$\frac{1}{10^n}+ \dots + \frac{1}{10^{n+1}-1} > \frac{9\times 10^n}{10^{n+1}-1} > \frac{9}{10} $
Cho $n $ chạy từ $0 $ đến $\infty $ ta sẽ có điều phải chứng minh
Cách này có sơ cấp không?
Sử dụng bất đẳng thức AM-GM ta có
$\frac{2}1+\frac{3}2+\frac{4}3+...+\frac{n+1}n\ge n\sqrt[n]{n+1} $
hay $(1+1)+(1+\frac{1}2)+(1+\frac{1}3)+...+(1+\frac{1}n )\ge n\sqrt[n]{n+1} $
nghĩa là $1+\frac1{2}+...+\frac1{n}\ge n(\sqrt[n]{n+1}-1) $
Tất nhiên $n(\sqrt[n]{n+1}-1)\rightarrow +\infty $ khi $n\rightarrow +\infty $. Xong!
[RIGHT][I][B]Nguồn: MathScope.ORG[/B][/I][/RIGHT]
 
__________________
Một chút cho tâm hồn bay xa
dduclam is offline   Trả Lời Với Trích Dẫn
The Following 2 Users Say Thank You to dduclam For This Useful Post:
huynhcongbang (12-08-2010), xtungftu (21-08-2011)
Old 12-08-2010, 09:51 PM   #6
herr.casanova
+Thành Viên+
 
Tham gia ngày: Aug 2010
Bài gởi: 33
Thanks: 17
Thanked 33 Times in 13 Posts
Trích:
Nguyên văn bởi dduclam View Post
Cách này có sơ cấp không?
Sử dụng bất đẳng thức AM-GM ta có
$\frac{2}1+\frac{3}2+\frac{4}3+...+\frac{n+1}n\ge n\sqrt[n]{n+1} $
hay $(1+1)+(1+\frac{1}2)+(1+\frac{1}3)+...+(1+\frac{1}n )\ge n\sqrt[n]{n+1} $
nghĩa là $1+\frac1{2}+...+\frac1{n}\ge n(\sqrt[n]{n+1}-1) $
Tất nhiên $n(\sqrt[n]{n+1}-1)\rightarrow +\infty $ khi $n\rightarrow +\infty $. Xong!
Phiền bạn chứng minh nốt $n(\sqrt[n]{n+1}-1)\rightarrow +\infty $
[RIGHT][I][B]Nguồn: MathScope.ORG[/B][/I][/RIGHT]
 
herr.casanova is offline   Trả Lời Với Trích Dẫn
Old 12-08-2010, 11:53 PM   #7
huynhcongbang
Administrator

 
huynhcongbang's Avatar
 
Tham gia ngày: Feb 2009
Đến từ: Ho Chi Minh City
Bài gởi: 2,413
Thanks: 2,165
Thanked 4,188 Times in 1,381 Posts
Gửi tin nhắn qua Yahoo chát tới huynhcongbang
Trích:
Nguyên văn bởi dduclam View Post
Tất nhiên $n(\sqrt[n]{n+1}-1)\rightarrow +\infty $ khi $n\rightarrow +\infty $. Xong!
Không biết anh dduclam có cách nào chứng minh kq trên ngắn gọn không nữa, em thử cách này!
Ta sẽ chứng minh rằng:
$lim(n\sqrt[n]{n}-n)=+\infty $
Thật vậy:
Trước hết, ta cần chứng minh:$lim\sqrt[n]{n}=1 $.
Ta xét:
$y=\sqrt[n]{n}\Rightarrow lny=\frac{lnn}{n}<\frac{\sqrt{n}}{n}=\frac{1}{ \sqrt{n}}\Rightarrow lim(lny)=0\Rightarrow limy=1 $
Ta có giới hạn cơ bản là:
$\lim_{x\rightarrow 0 }\frac{e^x-1}{x}=1\Rightarrow \lim_{n\rightarrow +\infty }\frac{e^{\frac{1}{n}}-1}{\frac{1}{n}}=1\Rightarrow \lim_{n\rightarrow +\infty }n(\sqrt[n]{e}-1)=1 $
Do đó:
$\lim n(\sqrt[n]{n}-1)=lim[n(\sqrt[n]{e}-1)].\frac{\sqrt[n]{n}-1}{\sqrt[n]{e}-1}=lim[n(\sqrt[n]{e}-1)].\frac{1-\frac{1}{\sqrt[n]{n}}}{\sqrt[n]{\frac{e}{n}}-\frac{1}{\sqrt[n]{n}}}=+\infty $
Đến đây mới có điều tất nhiên ở trên được!
[RIGHT][I][B]Nguồn: MathScope.ORG[/B][/I][/RIGHT]
 

thay đổi nội dung bởi: huynhcongbang, 13-08-2010 lúc 12:04 AM
huynhcongbang is offline   Trả Lời Với Trích Dẫn
The Following User Says Thank You to huynhcongbang For This Useful Post:
herr.casanova (13-08-2010)
Old 13-08-2010, 12:10 AM   #8
herr.casanova
+Thành Viên+
 
Tham gia ngày: Aug 2010
Bài gởi: 33
Thanks: 17
Thanked 33 Times in 13 Posts
Nếu thế thì cách của bạn dduclam không còn đơn giản nữa rồi D
[RIGHT][I][B]Nguồn: MathScope.ORG[/B][/I][/RIGHT]
 
herr.casanova is offline   Trả Lời Với Trích Dẫn
Old 13-08-2010, 12:47 AM   #9
dduclam
+Thành Viên Danh Dự+
 
dduclam's Avatar
 
Tham gia ngày: Nov 2007
Đến từ: Đại học Sư phạm Hà Nội
Bài gởi: 481
Thanks: 63
Thanked 168 Times in 92 Posts
Gửi tin nhắn qua Yahoo chát tới dduclam
Trích:
Nguyên văn bởi herr.casanova View Post
Nếu thế thì cách của bạn dduclam không còn đơn giản nữa rồi D
Ai bảo không đơn giản
Đặt $a_n=\sqrt[n]{n+1}-1>0 $ thì $(a_n+1)^n=n+1 $ hay $ln(n+1)=nln(a_n+1)<na_n \Rightarrow na_n\rightarrow+\infty $
xong chưa?
[RIGHT][I][B]Nguồn: MathScope.ORG[/B][/I][/RIGHT]
 
__________________
Một chút cho tâm hồn bay xa
dduclam is offline   Trả Lời Với Trích Dẫn
The Following 2 Users Say Thank You to dduclam For This Useful Post:
herr.casanova (13-08-2010), huynhcongbang (13-08-2010)
Old 13-08-2010, 02:11 AM   #10
herr.casanova
+Thành Viên+
 
Tham gia ngày: Aug 2010
Bài gởi: 33
Thanks: 17
Thanked 33 Times in 13 Posts
Trích:
Nguyên văn bởi dduclam View Post
Ai bảo không đơn giản
Đặt $a_n=\sqrt[n]{n+1}-1>0 $ thì $(a_n+1)^n=n+1 $ hay $ln(n+1)=nln(a_n+1)<na_n \Rightarrow na_n\rightarrow+\infty $
xong chưa?
Bạn chứng minh nốt $\ln{(a_n+1)} < a_n $.
Cái này tất nhiên không khó, nhưng bạn nên nói ra rõ ràng vì không phải ai cũng hiểu nhanh được
[RIGHT][I][B]Nguồn: MathScope.ORG[/B][/I][/RIGHT]
 

thay đổi nội dung bởi: herr.casanova, 13-08-2010 lúc 02:19 AM
herr.casanova is offline   Trả Lời Với Trích Dẫn
Old 13-08-2010, 06:02 AM   #11
Traum
Moderator
 
Traum's Avatar
 
Tham gia ngày: Nov 2007
Đến từ: cyber world
Bài gởi: 413
Thanks: 14
Thanked 466 Times in 171 Posts
$A = 1 + 1/2 + 1/3 + 1/4 + 1/5 +...= (1 + 1/3 + 1/5 + ...) + (1/2 + 1/4 + 1/6 +...) > (1/2 + 1/4 + 1/6 + ...) + (1/2 + 1/4 + 1/6 + ...) = 1 + 1/2 + 1/3 + 1/4 + 1/5 +... = A $ Tại sao ???
[RIGHT][I][B]Nguồn: MathScope.ORG[/B][/I][/RIGHT]
 
__________________
Traum is giấc mơ.

thay đổi nội dung bởi: Traum, 13-08-2010 lúc 02:16 PM
Traum is offline   Trả Lời Với Trích Dẫn
The Following User Says Thank You to Traum For This Useful Post:
herr.casanova (13-08-2010)
Old 13-08-2010, 01:50 PM   #12
herr.casanova
+Thành Viên+
 
Tham gia ngày: Aug 2010
Bài gởi: 33
Thanks: 17
Thanked 33 Times in 13 Posts
Quá chuẩn!!! Ý tưởng sáng tạo, bái phục bái phục Từ đây cũng suy ra được giới hạn là vô cùng

ps. cái dấu đầu tiên là dấu bằng chứ đâu phải dấu lớn hơn???
[RIGHT][I][B]Nguồn: MathScope.ORG[/B][/I][/RIGHT]
 
herr.casanova is offline   Trả Lời Với Trích Dẫn
Old 13-08-2010, 02:14 PM   #13
Traum
Moderator
 
Traum's Avatar
 
Tham gia ngày: Nov 2007
Đến từ: cyber world
Bài gởi: 413
Thanks: 14
Thanked 466 Times in 171 Posts
Ừ, đúng rồi. Thanks nhiều!
[RIGHT][I][B]Nguồn: MathScope.ORG[/B][/I][/RIGHT]
 
__________________
Traum is giấc mơ.
Traum is offline   Trả Lời Với Trích Dẫn
Old 14-08-2010, 03:58 AM   #14
dduclam
+Thành Viên Danh Dự+
 
dduclam's Avatar
 
Tham gia ngày: Nov 2007
Đến từ: Đại học Sư phạm Hà Nội
Bài gởi: 481
Thanks: 63
Thanked 168 Times in 92 Posts
Gửi tin nhắn qua Yahoo chát tới dduclam
Trích:
Nguyên văn bởi herr.casanova View Post
Bạn chứng minh nốt $\ln{(a_n+1)} < a_n $.
Cái này tất nhiên không khó, nhưng bạn nên nói ra rõ ràng vì không phải ai cũng hiểu nhanh được
Cái $\ln(1+x)<x\Leftrightarrow 1+x<e^x, \forall x>0 $ có thể chứng minh đơn giản bằng đạo hàm, nhưng ta thử tìm một chứng minh non-đạo hàm xem.
Theo Bernoulli $e^x=[1+(e-1)]^x \ge 1+(e-1)x > 1+x. $
[RIGHT][I][B]Nguồn: MathScope.ORG[/B][/I][/RIGHT]
 
__________________
Một chút cho tâm hồn bay xa
dduclam is offline   Trả Lời Với Trích Dẫn
Trả lời Gởi Ðề Tài Mới

Bookmarks

Ðiều Chỉnh
Xếp Bài

Quuyền Hạn Của Bạn
You may not post new threads
You may not post replies
You may not post attachments
You may not edit your posts

BB code is Mở
Smilies đang Mở
[IMG] đang Mở
HTML đang Tắt

Chuyển đến


Múi giờ GMT. Hiện tại là 12:43 PM.


Powered by: vBulletin Copyright ©2000-2024, Jelsoft Enterprises Ltd.
Inactive Reminders By mathscope.org
[page compression: 100.83 k/116.15 k (13.19%)]